LSAT and Law School Admissions Forum

Get expert LSAT preparation and law school admissions advice from PowerScore Test Preparation.

User avatar
 Dave Killoran
PowerScore Staff
  • PowerScore Staff
  • Posts: 5853
  • Joined: Mar 25, 2011
|
#46250
Complete Question Explanation
(The complete setup for this game can be found here: lsat/viewtopic.php?t=14384)

The correct answer choice is (E)

This extremely difficult question forces you to account for the variables remaining for team 2. In answer choices (A) and (C), the variables remaining for team 2 include pairs of variables which cannot go together according to the rules:

Answer choice (A): When G, K, and L are assigned to team 1, then H, N, P, and Q are the jugglers available to be assigned to team 2. But, from the fifth rule P and Q cannot be assigned to the same team, so the pool is actually H, N, and P/Q. However, if H is assigned to team 2, then Q must be assigned to the middle position on team 1 per the sixth rule. As that is not the case in this answer choice, H cannot be assigned to team 2. Consequently, there are not enough jugglers available for team 2, and this answer choice cannot occur.

Answer choice (C): When L, K, and Q are assigned to team 1, then G, H, N, and P are the jugglers available to be assigned to team 2. But, from the first rule G and H cannot be assigned to the same team, and from the fourth rule N and P cannot be assigned to the same team, so the pool is actually just G/H and N/P. Consequently, there are not enough jugglers available for team 2, and this answer choice cannot occur.

In answer choices (B) and (D), the variables remaining for team 2 include H and L, but as L cannot be on team 2, H must be on team 2, and assigning H to team 2 forces Q into the middle position on team 1, which does not appear in either answer choice. Thus, both answer choices are incorrect.

Answer choice (B): When G, K, and P are assigned to team 1, then H, L, N, and Q are the jugglers available to be assigned to team 2. But, from the third rule, L cannot be assigned to team 2, so the pool is actually H, N, and Q. However, if H is assigned to team 2, then Q must be assigned to the middle position on team 1. As that is not the case in this answer choice, H cannot be assigned to team 2. Consequently, there are not enough jugglers available for team 2, and this answer choice cannot occur.

Answer choice (D): When Q, K, and P are assigned to team 1, then G, H, L, and N are the jugglers available to be assigned to team 2. But, from the third rule, L cannot be assigned to team 2, so the pool is actually G, H, and N. However, if H is assigned to team 2, then Q must be assigned to the middle position on team 1. As that is not the case in this answer choice, H cannot be assigned to team 2. Consequently, there are not enough jugglers available for team 2, and this answer choice cannot occur.

Thus, as the first four answer choices have been eliminated, answer choice (E) must be correct.

Overall, this is a very hard question.
User avatar
 rbrar_71
  • Posts: 3
  • Joined: Nov 02, 2021
|
#91771
Hello,

I found it helpful that none of the answer choices contained Q in the middle. From this, you can infer that H can never be in the front row of team 2 and therefore is not in the game. This allowed me to use the remaining variables for team 2 in each answer choice along with the not-laws to easily eliminate answer choices A, B, C, and D.

Hope this is helpfuL!
User avatar
 atierney
PowerScore Staff
  • PowerScore Staff
  • Posts: 215
  • Joined: Jul 06, 2021
|
#91806
Awesome! Yes, that is a valid inference, and here it allowed to easily locate the correct answer. The games are a bit of a scavenger hunt; you are playing detective and gathering clues as you go (that were not able to be inferred during the initial set-up), and this active mindset is crucial to a high score on the Logic Game section.

Get the most out of your LSAT Prep Plus subscription.

Analyze and track your performance with our Testing and Analytics Package.